LSAT and Law School Admissions Forum

Get expert LSAT preparation and law school admissions advice from PowerScore Test Preparation.

User avatar
 Hanin Abu Amara
PowerScore Staff
  • PowerScore Staff
  • Posts: 60
  • Joined: Mar 29, 2023
|
#101273
Complete Question Explanation

Assumption. The correct answer choice is (A).

The author concludes that most viewers who watched the program the 1st night also watched it the second night.

The premise was: there was a small audience on Thursday for part one and a slightly smaller audience for part 2 on Friday,

In Assumption questions, we look at what assumption are absolutely necessary for the argument to make sense. Here there is a clear problem. Just because there is a smaller group doesn't mean it was the same viewers. What if new people tuned in for part 2.

We want to fix the problem here. So our prephrase is that most people that watched part 1 also watched part 2.

Answer choice (A): This is the correct answer choice. This matches our prephrase. We need it to be true that the audience was largely comprised of the same people.

Answer choice (B): It is not necessary that most of them liked it the second night since we are only concerned about who showed up to the second night.

Answer choice (C): Comparing the first night to the second is not necessary for the conclusion. If we negate it and said they enjoyed the second night more than the first then we didn't impact the conclusion.

Answer choice (D): The people who didn't tune in on one of the nights are irrelevant.

Answer choice (E): People who generally like documentaries doesn't tell us if they were the same people both nights
 jdfrias.jdf@gmail.com
  • Posts: 1
  • Joined: May 26, 2022
|
#96739
Hi, Just took the flex test and needed some more information on how we get to answer "A" for #2. Thanks in advance. I ultimately came down to option C and E but chose C instead. Thanks again.
User avatar
 katehos
PowerScore Staff
  • PowerScore Staff
  • Posts: 184
  • Joined: Mar 31, 2022
|
#96747
Hi jdfrias.jdf!

The argument in the stimulus concludes that most of the viewers who watched Train Stories on the first night liked it enough to watch it again on the second night. Why do they believe this?
 
      1) On the first night, the audience was small
      2) On the second night, the audience was not smaller

Right away, this should seem odd. How can one conclude that the viewers were mostly the same solely on the basis of the group size? There is a missing piece to this puzzle. This piece is exactly what we must find out - the assumption the argument made!

There are two types of assumptions: Supporter and Defender. Supporter assumptions link 'rogue' elements of the stimulus, while Defender assumptions eliminate possibilities that could harm the stimulus. Since we have a gap in logic, we can probably guess we are looking for a Supporter assumption. So, our prephrase can be any assumption that seals this gap.

Looking at answer choice (A), we see that most viewers who watched the second night also watched the first. Well, if that's the case, then this helps seal the gap we noticed earlier! If most of the people who watched the second night (which was not significantly smaller than the first) also watched the first night, then the claim that most of the viewers from the first night liked it enough to tune in the second night is supported. If we apply the Assumption Negation Technique to this answer choice, we can confirm it is correct. The negated version of (A) would be something along the lines of "Most of the viewers who tuned in the second night did NOT watch the program the first night." This negated version of the answer choice very clearly attacks the argument presented in the stimulus, so we know this is our correct answer!

Each of the other answer choices contain irrelevant or extraneous information. Consider this: does it matter whether the most of the viewers on the second night liked Train Stories? Does it matter whether they enjoyed it more the first time than the second? Nope! The same thing goes for the rest of the answer choices. Remember: Assumption answer choices cannot contain extraneous information, so if they do you can eliminate them.

I hope this helps :)
Kate
User avatar
 mkarimi73
  • Posts: 73
  • Joined: Aug 18, 2022
|
#97751
Did this question require any "math" or "abstraction" of the stimulus to understand what is going on? I got the answer right purely by the Negation technique and POE, but I'd like to know essentially the math behind this question, if there is any. Thanks.
User avatar
 atierney
PowerScore Staff
  • PowerScore Staff
  • Posts: 215
  • Joined: Jul 06, 2021
|
#98102
I wouldn't call what you would need to do here "math," but if you want to look "abstractly," you basically have two populations X (first day viewers) and Y (second day viewers). Now, without more, these are two independent variables (populations); however, the idea is that it's likely (but not necessary) that these are not completely independent, and indeed, it would be reasonable to assume that X and Y overlap significantly (think of a Venn Diagram).

Like I said, although we know that Y is less than X, there isn't a lot of "math" to do. Just the understanding of the likely assumption made in the argument (that is X and Y are significantly related), and then the necessity of the truth of that assumption.
User avatar
 maedayoku
  • Posts: 7
  • Joined: Sep 27, 2023
|
#103408
katehos wrote: Mon Aug 15, 2022 12:51 pm Hi jdfrias.jdf!

The argument in the stimulus concludes that most of the viewers who watched Train Stories on the first night liked it enough to watch it again on the second night. Why do they believe this?
 
      1) On the first night, the audience was small
      2) On the second night, the audience was not smaller

Right away, this should seem odd. How can one conclude that the viewers were mostly the same solely on the basis of the group size? There is a missing piece to this puzzle. This piece is exactly what we must find out - the assumption the argument made!

There are two types of assumptions: Supporter and Defender. Supporter assumptions link 'rogue' elements of the stimulus, while Defender assumptions eliminate possibilities that could harm the stimulus. Since we have a gap in logic, we can probably guess we are looking for a Supporter assumption. So, our prephrase can be any assumption that seals this gap.

Looking at answer choice (A), we see that most viewers who watched the second night also watched the first. Well, if that's the case, then this helps seal the gap we noticed earlier! If most of the people who watched the second night (which was not significantly smaller than the first) also watched the first night, then the claim that most of the viewers from the first night liked it enough to tune in the second night is supported. If we apply the Assumption Negation Technique to this answer choice, we can confirm it is correct. The negated version of (A) would be something along the lines of "Most of the viewers who tuned in the second night did NOT watch the program the first night." This negated version of the answer choice very clearly attacks the argument presented in the stimulus, so we know this is our correct answer!

Each of the other answer choices contain irrelevant or extraneous information. Consider this: does it matter whether the most of the viewers on the second night liked Train Stories? Does it matter whether they enjoyed it more the first time than the second? Nope! The same thing goes for the rest of the answer choices. Remember: Assumption answer choices cannot contain extraneous information, so if they do you can eliminate them.

I hope this helps :)
Kate
Hi Kate! I am just confused at why the negated version of A is "Most of the viewers who tuned in the second night did NOT watch the program the first night", rather than "LESS THAN HALF of the viewers who tuned in the second night had watched it the first night"?
User avatar
 Jeff Wren
PowerScore Staff
  • PowerScore Staff
  • Posts: 389
  • Joined: Oct 19, 2022
|
#103494
Hi maedayoku,

The definition of the word "most" is "more than half."

The problem with using the phrase "less than half" here is that you're actually missing the possibility of a perfect 50/50 split. If exactly half of the viewers who watched the second night had also watched the first night, that would not fall under "most" or under "less than half."

The correct logical opposite term of "most" would be "50% or less" rather than "less than half." Of course, it's probably easier to just say "most second night viewers did not watch the first night" which is logically equivalent to saying 50% or less did watch both nights.

Logical Opposition is discussed in more detail in lesson 2 of The PowerScore LSAT Course and in chapter 11 of "The Logical Reasoning Bible."
User avatar
 maedayoku
  • Posts: 7
  • Joined: Sep 27, 2023
|
#103554
Jeff Wren wrote: Mon Oct 02, 2023 1:16 pm Hi maedayoku,

The definition of the word "most" is "more than half."

The problem with using the phrase "less than half" here is that you're actually missing the possibility of a perfect 50/50 split. If exactly half of the viewers who watched the second night had also watched the first night, that would not fall under "most" or under "less than half."

The correct logical opposite term of "most" would be "50% or less" rather than "less than half." Of course, it's probably easier to just say "most second night viewers did not watch the first night" which is logically equivalent to saying 50% or less did watch both nights.

Logical Opposition is discussed in more detail in lesson 2 of The PowerScore LSAT Course and in chapter 11 of "The Logical Reasoning Bible."
Thank you so much! It really clears it up!

Get the most out of your LSAT Prep Plus subscription.

Analyze and track your performance with our Testing and Analytics Package.